You are on page 1of 3

Adam Taylor

MAP 6505
Extra credit problem:
n

i) Use the Fourier Transform with respect to


Schrodinger equation:

to find the distributional solution to the IVP for the

1
itU t ( x) xU t ( x),
2

lim U t ( x) ( x)

t 0

Solution to part (i):


Taking the Fourier transform of the equations, we see that

i t F[U t ](k )
This is an ODE with solution F[U t ](k ) e

F[U t ](k ),

it k

U t ( x ) F [e
1

F[U 0 ](k ) 1

. Then the solution to the IVP is

it k

1
]( x)
2

it k

ei ( k , x ) dk

By Fubinis theorem, this can be written as

1
2

tk 2
i k1 x1 1

dk1 e

tk 2
i k2 x2 2

dk2 ... e

tk 2
i kn xn n

dkn

We calculate

tk 2
i km xm m

dkm

2
i t

q2 i

2
qxm
t

dq

2
i t

ix 2
m
2t

i
q
xm
2 t

2
dq
e
it

ix 2m
2t

1 2 2 2t
Then U t ( x)
.

e
2 it
ix

ii) Prove that the solution to the Schrodinger problem describing propagation of the initial wave packet
0 ( x) ,

1
it ( x, t ) x ( x, t ), ( x, 0) 0 ( x)
2
1

is given by the convolution in the variable x : ( x, t ) U t 0 ( x)

Solution to part (ii):


Considering that lim U t ( x) ( x) , we see that ( x,0) 0 ( x) 0 ( x) . Then the proposed
t 0

solution satisfied the initial conditions. Note also that i tU t ( x)

1
xU t ( x) 0 . Then
2

1
1
i t ( x, t ) x ( x, t ) i t U t 0 ( x) x U t 0 ( x)
2
2
1
i tU t 0 ( x) xU t 0 ( x)
2
1
i tU t ( x) 0 ( x )d xU t ( x) 0 ( x )d
2 x
x

i tU t ( x) xU t ( x) 0 ( x )d 0
2

Then the equation is satisfied by ( x, t ) U t 0 ( x) .


iii) Investigate the limit of the probability to find the particle in the initial ball

P(t )

( x, t ) dx
2

x R

as t .

Solution to part (iii):


We write
n

1 2 2
( x, t )

2 it
We first note that

i x
2t

0 ( )d

( ) d 1 . Since the integrand is bounded, we can us the Lebesque dominated

convergence theorem to see that

lim
t

i x
2t

0 ( )d

( )d

1
Since this quantity is bounded, it is clear that ( x, t ) 0 with
2
as t .

2 2

as t . Then P(t ) 0
it

Comments: It is clear that the solution Schrodinger equation violates the assumption of relativity theory
that no signal can propagate faster than the speed of light. Letting 0 CR ( R x ) and choosing CR
such that P(0) 1 , we see that

CR

( x, t )

CR

1 2 2

2 it

i x
2t

dx

x R

dx

x R

1 2 2

2 it

i x
2t

While supp ( x,0) 0 ( x) BR , for any t 0 the function ( x, t ) , and hence ( x, t ) , has its
2

support in the whole space. This can be physically interpreted by stating that in any period of time, there
is finite probability that the particle has travelled to anywhere in the infinite space, meaning perhaps
farther than light can have possibly travelled according the principles of modern physics.

You might also like